Diễn Đàn MathScopeDiễn Đàn MathScope
  Diễn Đàn MathScope
Ghi Danh Hỏi/Ðáp Community Lịch

Go Back   Diễn Đàn MathScope > Sơ Cấp > Đại Số và Lượng Giác > Các Bài Toán Đã Được Giải

News & Announcements

Ngoài một số quy định đã được nêu trong phần Quy định của Ghi Danh , mọi người tranh thủ bỏ ra 5 phút để đọc thêm một số Quy định sau để khỏi bị treo nick ở MathScope nhé !

* Nội quy MathScope.Org

* Một số quy định chung !

* Quy định về việc viết bài trong diễn đàn MathScope

* Nếu bạn muốn gia nhập đội ngũ BQT thì vui lòng tham gia tại đây

* Những câu hỏi thường gặp

* Về việc viết bài trong Box Đại học và Sau đại học


Ðề tài đã khoá Gởi Ðề Tài Mới
 
Ðiều Chỉnh Xếp Bài
Old 29-06-2011, 12:29 PM   #1531
khtoan
+Thành Viên+
 
Tham gia ngày: Jan 2010
Đến từ: Đà Nẵng
Bài gởi: 155
Thanks: 23
Thanked 128 Times in 68 Posts
Trích:
Nguyên văn bởi kid3494 View Post
Cho $a, b, c > 0 $
CMR $\frac{a}{b + 2} + \frac{b}{c + 2} + \frac{c}{a + 2} \leq 1 $
Bài này mình nhớ đk là $a^2+b^2+c^2=3 $
[RIGHT][I][B]Nguồn: MathScope.ORG[/B][/I][/RIGHT]
 
khtoan is offline  
Old 29-06-2011, 03:56 PM   #1532
ilovehien95
+Thành Viên+
 
Tham gia ngày: Jun 2011
Bài gởi: 76
Thanks: 142
Thanked 13 Times in 8 Posts
Trích:
Nguyên văn bởi khtoan View Post
Theo cosi: $ab+bc+ca\geqslant 3 $
$\sum \frac{a}{\sqrt{b^{2}+3}}\geqslant \sum \frac{a}{\sqrt{b^2+ab+bc+ca}}=\sum \frac{a}{\sqrt{(b+a)(b+c)}}\geqslant \sum \frac{2a}{a+2b+c}\geqslant \frac{2(a+b+c)^2}{a^2+b^2+c^2+3ab+3bc+3ca} \geqslant \frac{3}{2} $
Đẳng thức xảy ra khi và chỉ khi a=b=c=1
đẳng thức thứ 2 sử dụng ngược dấu rồi
------------------------------
Trích:
Nguyên văn bởi Nguyenhuyen_AG View Post
Ta chỉ cần chứng minh được
$\sqrt{\frac{a}{b+3}}+\sqrt{\frac{b}{c+3}}+ \sqrt{\frac{c}{a+3}}\geq \frac{3}{2} $
Thay bộ $(a,b,c) $ bởi $\left ( \frac{x}{y},\frac{z}{x},\frac{y}{z} \right ). $ Ta đưa bất đẳng thức về dạng đồng bậc là
$\frac{x}{\sqrt{3xy+yz}}+\frac{y}{\sqrt{3yz+zx}}+ \frac{z}{\sqrt{3zx+xy}}\ge\frac{3}{2} $
Sử dụng bất đẳng thức Holder, ta có
$\left (\sum \frac{x}{\sqrt{3xy+yz}} \right )^2\left [ \sum x(3xy+yz) \right ]\ge(x+y+z)^3 $
Vậy, ta cần chứng minh được
$(x+y+z)^3\ge \frac{27}{4}(x^2y+y^2z+z^2x+xyz) $
Đây là một kết quả quen thuộc.
anh Huyện cm dùm em bài toán quen thuộc đi. Bài này ko đối xứng khó làm quá
[RIGHT][I][B]Nguồn: MathScope.ORG[/B][/I][/RIGHT]
 
__________________
Listen to the rhymth of the falling rain. Tellling me what a fool i've been........I CANT love another when my heart somewhere faraway

thay đổi nội dung bởi: ilovehien95, 29-06-2011 lúc 03:57 PM Lý do: Tự động gộp bài
ilovehien95 is offline  
Old 29-06-2011, 04:20 PM   #1533
toanlc_gift
+Thành Viên+
 
toanlc_gift's Avatar
 
Tham gia ngày: Jan 2009
Đến từ: FU
Bài gởi: 171
Thanks: 31
Thanked 142 Times in 80 Posts
Trích:
Bài toán mạnh hơn sau đây vẫn đúng
$\frac{a}{{\sqrt {{b^2} + 8} }} + \frac{b}{{\sqrt {{c^2} + 8} }} + \frac{c}{{\sqrt {{a^2} + 8} }} \ge 1 $
với là các số dương có tích bằng 1
Đặt ${a^2};{b^2};{c^2} $bằng ${x^3};{y^3};{z^3} $,khi đó $xyz=1 $
bđt trở thành
$\sqrt {\frac{{{x^3}}}{{{y^3} + 8}}} + \sqrt {\frac{{{y^3}}}{{{z^3} + 8}}} + \sqrt {\frac{{{z^3}}}{{{x^3} + 8}}} \ge 1 $
sử dụng Schwarz và cauchyschwarz:
$VT = \frac{{{x^2}}}{{\sqrt {xy} \sqrt {{y^2} + 8xz} }} + \frac{{{y^2}}}{{\sqrt {yz} \sqrt {({z^2} + 8xy)} }} + \frac{{{z^2}}}{{\sqrt {zx} \sqrt {{x^2} + 8yz} }} $
$\ge \frac{{{{(x + y + z)}^2}}}{{\sqrt {xy} \sqrt {{y^2} + 8xz} + \sqrt {yz} \sqrt {({z^2} + 8xy)} + \sqrt {zx} \sqrt {{x^2} + 8yz} }} $
$\ge \frac{{{{(x + y + z)}^2}}}{{\sqrt {(xy + yz + zx)({x^2} + {y^2} + {z^2} + 8(xy + yz + zx))} }} $
$\ge \frac{{{{(x + y + z)}^2}}}{{\sqrt {\frac{{{{(x + y + z)}^2}}}{3}({{(x + y + z)}^2} + 2{{(x + y + z)}^2})} }} = 1 $
vậy bđt được chứng minh
bài toán ban đầu có thể chứng minh tương tự
------------------------------------------------------------------------
Ta có thể chứng minh được bài toán với mọi $k \ge 0 $
$\frac{a}{{\sqrt {{b^2} + k} }} + \frac{b}{{\sqrt {{c^2} + k} }} + \frac{c}{{\sqrt {{a^2} + k} }} \ge \frac{3}{{\sqrt {k + 1} }} $
với $k \ge 1 $ sử dụng cách trên(chú ý đoạn cuối dùng AM-GM dưới mẫu để đưa về tổng)
với $0 \le k \le 1 $,đổi biến nghịch đảo sẽ đưa được về trường hợp $k \ge 1 $
[RIGHT][I][B]Nguồn: MathScope.ORG[/B][/I][/RIGHT]
 

thay đổi nội dung bởi: toanlc_gift, 29-06-2011 lúc 04:57 PM Lý do: sửa bài đâu cần lý do ^^!
toanlc_gift is offline  
The Following User Says Thank You to toanlc_gift For This Useful Post:
Mr_Trang (30-06-2011)
Old 29-06-2011, 04:47 PM   #1534
daiduong1095
+Thành Viên+
 
daiduong1095's Avatar
 
Tham gia ngày: Sep 2010
Đến từ: CVP-Math
Bài gởi: 287
Thanks: 13
Thanked 210 Times in 112 Posts
Gửi tin nhắn qua Yahoo chát tới daiduong1095
Trích:
Nguyên văn bởi ilovehien95 View Post
đẳng thức thứ 2 sử dụng ngược dấu rồi
------------------------------

anh Huyện cm dùm em bài toán quen thuộc đi. Bài này ko đối xứng khó làm quá
Bài này quen thật mà bạn.Giả sử z là số không lớn nhất không nhỏ nhất.
Ta có:$(z-x)(z-y)\leq0\Rightarrow x^{2}y+y^{2}z+z^{2}x+xyz\leq z\left(x+y \right)^{2}\leq\frac{4}{27}\left(z+2.\frac{x+y}{2} \right)^{3} $
$=\frac{4}{27}\left(x+y+z \right)^{3} $.
[RIGHT][I][B]Nguồn: MathScope.ORG[/B][/I][/RIGHT]
 
__________________

thay đổi nội dung bởi: daiduong1095, 29-06-2011 lúc 04:51 PM
daiduong1095 is offline  
The Following User Says Thank You to daiduong1095 For This Useful Post:
ilovehien95 (02-07-2011)
Old 29-06-2011, 06:33 PM   #1535
bboy114crew
+Thành Viên+
 
Tham gia ngày: Oct 2010
Đến từ: Dòng thời gian...
Bài gởi: 294
Thanks: 290
Thanked 189 Times in 91 Posts
Gửi tin nhắn qua Yahoo chát tới bboy114crew
USAMO:
cho $0 < a \le b \le c \le d $ CMR
${a^b}{b^c}{c^d}{d^a} \ge {a^d}{d^c}{c^b}{b^a} $
[RIGHT][I][B]Nguồn: MathScope.ORG[/B][/I][/RIGHT]
 
__________________
Thay đổi tất cả và mãi mãi......
Offline...
bboy114crew is offline  
Old 29-06-2011, 09:06 PM   #1536
khtoan
+Thành Viên+
 
Tham gia ngày: Jan 2010
Đến từ: Đà Nẵng
Bài gởi: 155
Thanks: 23
Thanked 128 Times in 68 Posts
[QUOTE=ilovehien95;103284]đẳng thức thứ 2 sử dụng ngược dấu rồi
------------------------------
Ngược dấu chỗ nào bạn
[RIGHT][I][B]Nguồn: MathScope.ORG[/B][/I][/RIGHT]
 
khtoan is offline  
Old 29-06-2011, 10:47 PM   #1537
kid3494
+Thành Viên+
 
kid3494's Avatar
 
Tham gia ngày: Mar 2011
Bài gởi: 53
Thanks: 31
Thanked 9 Times in 7 Posts
Các bạn giúp mình bài tập này với
Cho $a, b, c > 0 $
CMR $\frac{2(a^{3} + b^{3} + c^{3})}{abc} + \frac{9(a + b + c)^{2}}{a^{2} + b^{2} + c^{2}} \geq 33 $
[RIGHT][I][B]Nguồn: MathScope.ORG[/B][/I][/RIGHT]
 
kid3494 is offline  
Old 29-06-2011, 11:16 PM   #1538
daiduong1095
+Thành Viên+
 
daiduong1095's Avatar
 
Tham gia ngày: Sep 2010
Đến từ: CVP-Math
Bài gởi: 287
Thanks: 13
Thanked 210 Times in 112 Posts
Gửi tin nhắn qua Yahoo chát tới daiduong1095
Trích:
Nguyên văn bởi kid3494 View Post
Các bạn giúp mình bài tập này với
Cho $a, b, c > 0 $
CMR $\frac{2(a^{3} + b^{3} + c^{3})}{abc} + \frac{9(a + b + c)^{2}}{a^{2} + b^{2} + c^{2}} \geq 33 $
Bài này mình làm như sau:
BDT tương đương với:$\frac{2\left(a^{3} + b^{3} + c^{3}-3abc \right)}{abc}\geq 9\frac{3(a^{2} + b^{2} + c^{2})-(a+b+c)^{2}}{a^{2} + b^{2} + c^{2}} $
$\Leftrightarrow (a^2+b^2+c^2-ab-bc-ca)\left(\frac{a+b+c}{abc}-\frac{9}{a^2+b^2+c^2} \right)\geq0 $.
$\Leftrightarrow (a^2+b^2+c^2-ab-bc-ca)\left((a+b+c)(a^2+b^2+c^2)-9abc \right)\geq0 $.Hiển nhiên!
[RIGHT][I][B]Nguồn: MathScope.ORG[/B][/I][/RIGHT]
 
__________________

thay đổi nội dung bởi: daiduong1095, 29-06-2011 lúc 11:29 PM
daiduong1095 is offline  
The Following User Says Thank You to daiduong1095 For This Useful Post:
anh_96 (30-06-2011)
Old 29-06-2011, 11:22 PM   #1539
kid3494
+Thành Viên+
 
kid3494's Avatar
 
Tham gia ngày: Mar 2011
Bài gởi: 53
Thanks: 31
Thanked 9 Times in 7 Posts
Trích:
Nguyên văn bởi daiduong1095 View Post
Bài này mình làm như sau:
BDT tương đương với:$\frac{2\left(a^{3} + b^{3} + c^{3}-3abc \right)}{abc}\geq 9\frac{3(a^{2} + b^{2} + c^{2})-(a+b+c)^{2}}{a^{2} + b^{2} + c^{2}} $
Cách này mình đã nghĩ ra, nhưng thầy nói có 2 cách. Không biết còn bạn nào có cách 2.
[RIGHT][I][B]Nguồn: MathScope.ORG[/B][/I][/RIGHT]
 
kid3494 is offline  
Old 29-06-2011, 11:26 PM   #1540
daiduong1095
+Thành Viên+
 
daiduong1095's Avatar
 
Tham gia ngày: Sep 2010
Đến từ: CVP-Math
Bài gởi: 287
Thanks: 13
Thanked 210 Times in 112 Posts
Gửi tin nhắn qua Yahoo chát tới daiduong1095
Trích:
Nguyên văn bởi kid3494 View Post
Cách này mình đã nghĩ ra, nhưng thầy nói có 2 cách. Không biết còn bạn nào có cách 2.
Bạn có thế đặt dùng p,q,r là cách 2 cũng ngắn.
[RIGHT][I][B]Nguồn: MathScope.ORG[/B][/I][/RIGHT]
 
__________________
daiduong1095 is offline  
Old 29-06-2011, 11:30 PM   #1541
th2091
+Thành Viên+
 
th2091's Avatar
 
Tham gia ngày: Sep 2010
Đến từ: THPT Kiến Thụy-Hải Phòng
Bài gởi: 140
Thanks: 39
Thanked 92 Times in 58 Posts
Trích:
Nguyên văn bởi kid3494 View Post
Các bạn giúp mình bài tập này với
Cho $a, b, c > 0 $
CMR $\frac{2(a^{3} + b^{3} + c^{3})}{abc} + \frac{9(a + b + c)^{2}}{a^{2} + b^{2} + c^{2}} \geq 33 $
Đặt p=a+b+c, q=ab+bc+ac, r=abc ta được
$2\frac{p^3-3pq+3r}{r}+9\frac{p^2}{p^2-2q}\ge 33 \Leftrightarrow 2\frac{p(p^2-3q)}{r}+9\frac{p^2}{p^2-2q}\ge 27 $
Ta có $r\le \frac{pq}{9} $ nên $LHS\ge 18\frac{p^2-3q}{q}+9(1+\frac{2q}{p^2-2q} $
Ta sẽ chứng minh $ 18\frac{p^2-3q}{q}+9(1+\frac{2q}{p^2-2q}\ge 27 \Leftrightarrow \frac{p^2}{q}+\frac{p}{p^2-2q}\ge 4 \Leftrightarrow (p^2-3q)^2\ge 0 $ (đúng)
[RIGHT][I][B]Nguồn: MathScope.ORG[/B][/I][/RIGHT]
 
th2091 is offline  
Old 29-06-2011, 11:47 PM   #1542
daiduong1095
+Thành Viên+
 
daiduong1095's Avatar
 
Tham gia ngày: Sep 2010
Đến từ: CVP-Math
Bài gởi: 287
Thanks: 13
Thanked 210 Times in 112 Posts
Gửi tin nhắn qua Yahoo chát tới daiduong1095
Trích:
Nguyên văn bởi kid3494 View Post
Các bạn giúp mình bài tập này với
Cho $a, b, c > 0 $
CMR $\frac{2(a^{3} + b^{3} + c^{3})}{abc} + \frac{9(a + b + c)^{2}}{a^{2} + b^{2} + c^{2}} \geq 33 $
Thêm bài tương tự :Cho a,b,c là các số thực dương.Cmr:
$\frac{a+b+c}{\sqrt[3]{abc}}+\frac{8abc}{(a+b)(b+c)(c+a)}\geq4 $.



[RIGHT][I][B]Nguồn: MathScope.ORG[/B][/I][/RIGHT]
 
__________________
daiduong1095 is offline  
Old 29-06-2011, 11:53 PM   #1543
Nguyenhuyen_AG
+Thành Viên+
 
Nguyenhuyen_AG's Avatar
 
Tham gia ngày: Apr 2010
Bài gởi: 300
Thanks: 35
Thanked 307 Times in 151 Posts
Trích:
Nguyên văn bởi daiduong1095 View Post
Thêm bài tương tự :Cho a,b,c là các số thực dương.Cmr:
$\frac{a+b+c}{\sqrt[3]{abc}}+\frac{8abc}{(a+b)(b+c)(c+a)}\geq4 $.


Bất đẳng thức có thể viết lại như sau
$\frac{a+b}{2\sqrt[3]{abc}}+\frac{b+c}{2\sqrt[3]{abc}}+\frac{c+a}{2\sqrt[3]{abc}}+\frac{8abc}{(a+b)(b+c)(c+a)}\geq4 $
Hiển nhiên đúng theo bất đẳng thức AM-GM.
Ngoài ra bất đẳng thức mạnh hơn sau đây vẫn đúng

$\frac{a+b+c}{\sqrt[3]{abc}}+\frac{4abc}{a^2b+b^2c+c^2a+abc}\geq4 $

[RIGHT][I][B]Nguồn: MathScope.ORG[/B][/I][/RIGHT]
 
__________________
Nguyen Van Huyen
Ho Chi Minh City University of Transport

thay đổi nội dung bởi: Nguyenhuyen_AG, 29-06-2011 lúc 11:56 PM
Nguyenhuyen_AG is offline  
The Following 2 Users Say Thank You to Nguyenhuyen_AG For This Useful Post:
bboy114crew (30-06-2011), ilovehien95 (30-06-2011)
Old 30-06-2011, 12:07 AM   #1544
daiduong1095
+Thành Viên+
 
daiduong1095's Avatar
 
Tham gia ngày: Sep 2010
Đến từ: CVP-Math
Bài gởi: 287
Thanks: 13
Thanked 210 Times in 112 Posts
Gửi tin nhắn qua Yahoo chát tới daiduong1095
Trích:
Nguyên văn bởi Nguyenhuyen_AG View Post
Bất đẳng thức có thể viết lại như sau
$\frac{a+b}{2\sqrt[3]{abc}}+\frac{b+c}{2\sqrt[3]{abc}}+\frac{c+a}{2\sqrt[3]{abc}}+\frac{8abc}{(a+b)(b+c)(c+a)}\geq4 $
Hiển nhiên đúng theo bất đẳng thức AM-GM.
Ngoài ra bất đẳng thức mạnh hơn sau đây vẫn đúng

$\frac{a+b+c}{\sqrt[3]{abc}}+\frac{4abc}{a^2b+b^2c+c^2a+abc}\geq4 $
Đúng ý mình đó .Thêm bài này cho các bạn tham khảo

Cho $a,b,c\geq0 $ và $a+b+c=3 $.Cmr:
$\frac{a+1}{b+1}+\frac{b+1}{c+1}+\frac{c+1}{a+1}\le \frac{25}{3\sqrt[3]{4ab+4bc+4ca}} $ .
[RIGHT][I][B]Nguồn: MathScope.ORG[/B][/I][/RIGHT]
 
__________________

thay đổi nội dung bởi: daiduong1095, 30-06-2011 lúc 12:09 AM
daiduong1095 is offline  
The Following User Says Thank You to daiduong1095 For This Useful Post:
ilovehien95 (30-06-2011)
Old 30-06-2011, 12:20 AM   #1545
Nguyenhuyen_AG
+Thành Viên+
 
Nguyenhuyen_AG's Avatar
 
Tham gia ngày: Apr 2010
Bài gởi: 300
Thanks: 35
Thanked 307 Times in 151 Posts
Trích:
Nguyên văn bởi daiduong1095 View Post
Đúng ý mình đó .Thêm bài này cho các bạn tham khảo

Cho $a,b,c\geq0 $ và $a+b+c=3 $.Cmr:
$\frac{a+1}{b+1}+\frac{b+1}{c+1}+\frac{c+1}{a+1}\le \frac{25}{3\sqrt[3]{4ab+4bc+4ca}} $ .
Sử dụng bất đẳng thức AM-GM, ta có

$3\sqrt[3]{4(ab+bc+ca)} \le 2+2+(ab+bc+ca)\le abc+ab+bc+ca+a+b+c+1=(a+1)(b+1)(c+1). $
Vậy, ta cần chứng minh được

$\frac{a+1}{b+1}+\frac{b+1}{c+1}+\frac{c+1}{a+1}\le \frac{25}{(a+1)(b+1)(c+1)} $

Hay là

$(a+1)^2(c+1)+(b+1)^2(a+1)+(c+1)^2(b+1)\le 25 $

$ab^2+bc^2+ca^2+(a+b+c)^2+3(a+b+c)+3\le 25 $

$ab^2+bc^2+ca^2\le 4 $

Thế nhưng đây là một kết quả quen thuộc. Đẳng thức xảy ra khi $(a,b,c) $ là một hoán vị của $(0,1,2). $
[RIGHT][I][B]Nguồn: MathScope.ORG[/B][/I][/RIGHT]
 
__________________
Nguyen Van Huyen
Ho Chi Minh City University of Transport

thay đổi nội dung bởi: Nguyenhuyen_AG, 30-06-2011 lúc 12:32 AM
Nguyenhuyen_AG is offline  
The Following User Says Thank You to Nguyenhuyen_AG For This Useful Post:
MathForLife (30-06-2011)
Ðề tài đã khoá Gởi Ðề Tài Mới

Bookmarks

Tags
bất đẳng thức


Quuyền Hạn Của Bạn
You may not post new threads
You may not post replies
You may not post attachments
You may not edit your posts

BB code is Mở
Smilies đang Mở
[IMG] đang Mở
HTML đang Tắt

Chuyển đến


Múi giờ GMT. Hiện tại là 05:59 AM.


Powered by: vBulletin Copyright ©2000-2024, Jelsoft Enterprises Ltd.
Inactive Reminders By mathscope.org
[page compression: 112.11 k/129.08 k (13.15%)]